Người ta dự định chuyển 78452 lít dầu từ kho A sang kho B Trong 4 ngày đầu mỗi ngày chuyển được 18214 lít dầu hỏi còn phải chuyển bao nhiêu lít dầu nữa

Answers

Answer 1

Answer:

5596 liters  of oil still need to be transferred.

Step-by-step explanation:

Total oil = 78452 liters

oil transferred each day for 4 days = 18214 liters

The amount of oil transferred in 4 days = 4 x 18214 = 72856 liters

The mount of oil left in ware house A

= 78452 - 72856

= 5596 liters  


Related Questions

The local amusement park was interested in the average wait time at their most popular roller coaster at the peak park time (2 p.m.). They selected 13 patrons and had them get in line between 2 and 3 p.m. Each was given a stopwatch to record the time they spent in line. The times recorded were as follows (in minutes): 118, 124, 108, 116, 99, 120, 148, 118, 119, 121, 45, 130, 118. What is the mode

Answers

Answer:

118

Step-by-step explanation:

Mode refers to a value that appears most frequently in a data set.

For example the mode in the data set below is 3 :

1, 2,3 4,3,3,9,10

3 is the mode because it appears 3 times while others appear once in the data set while the other values appear only once.

Mode is a measure of central tendency

118 appears 3 times while others appear once so it is the mode

3x-y=-1. X-2y=-12 solve by using graphing

Answers

Answer:

x = 2

y = 7

Step-by-step explanation:

Graphing calculators are everywhere but ok.

I attached a graph.

Kadeesha has a point card for a movie theater.
She receives 50 rewards points just for signing up she earns 12.5 points for each visit to the movie theater she needs at least 210 points for a free movie ticket right and solve for an inequality the number of visits kadeesha can make to earn her first free movie ticket

Answers

Answer:

50 + 12.5 v ≥ 210

She needs at least 13 visits

Step-by-step explanation:

points for signing up + points per visit* number of visits ≥  210

50 + 12.5 v ≥ 210

Subtract 50 from each side

50 + 12.5 v -50 ≥ 210-50

12.5 v ≥ 160

Divide by 12.5

12.5 v/12.5 ≥ 160/12.5

v ≥ 12.8

She needs at least 13 visits

Answer:

Step-by-step explanation:

Un deposito de agua tiene forma de un cubo de 10 metros de arista ¿cual es la capacidad del deposito?

Answers

Respuesta:

1000m³

Explicación paso a paso:

La capacidad del tanque viene dada por el volumen del cubo;

El volumen, V del cubo se da como: a³

V = a³

El borde del cubo mide 10 metros.

Por eso,

Volumen = 10³

Volumen = 10 * 10 * 10

Volumen = 1000 m³

Por lo tanto, la capacidad del tanque de agua es de 1000 m³.

-40 divided by what makes -4

Answers

Answer:

10

Step-by-step explanation:

<3

If f(x) = x3 + 5, then what is the remainder when f(x) is divided by x + 3?

Answers

Answer: -22

==========================================================

Explanation:

You could use synthetic division or polynomial long division to find the answer, but a much quicker way is to apply the remainder theorem.

That theorem says that if we divide p(x) over (x-k), then the remainder is p(k).

We're dividing over x+3, or x-(-3), so we can see that k = -3 in this case.

Plug this into the function

f(x) = x^3+5

f(-3) = (-3)^3 + 5

f(-3) = -27 + 5

f(-3) = -22 is the remainder

------------

If you want to use polynomial long division or synthetic division, then check out the diagram below.

What’s the answer?? Pls help

Answers

Answer:

11

Step-by-step explanation:

w

I need help with this question

Answers

Answer:

$18.63

Step-by-step explanation:

15% of 45 is equal to 45 times .15, which is equal to 6.75.

45+6.75 = 51.75

Ben resold the guitar for $51.75

36% of 51.75 is equal to 51.75*.36 or 18.63

Ben spent $18.63 on books

18.63

15% of 45 is equal to 45 times 15 which is equal to 6.75

Question and choices are in the photo please explain the answer

Answers

Answer:

336

Step-by-step explanation:

The exclamation mark in math represents the factorial operation, which means you multiply the number by decreasing positive integers.

Therefore:

5! = 5 x 4 x 3 x 2 x 1 = 120.

8! = 8 x 7 x 6 x 5 x 4 x 3 x 2 x 1 = 40320

8!/5! = 40320/120

40320/120 = 336

Which number could represent point Z

Answers

10 1/2 would be the place u dotted Z down

Each number in the sequence is 8 less than the previous number which list of number can be described by the following rule


A.69,60,53

B.53,61,69

C.53,60,69

D.69,61,53

Answers

Answer: a

Step-by-step explanation:

Factor the polynomial 3X² + 5X - 2.

Answers

Answer:

(x+2)(3x-1)

Step-by-step explanation:

3x^2 + 5x -2

we need two numbers which gives 6 when multiplied and 5 when subtracted.

so the numbers are 6 and 1 because 6*1=6 and 6-1 =5.S0,

3x^2 + (6-1)x -2

3x^2 +6x -x -2

take common from first two terms and from third and fourth term

3x(x+2) -1(x+2)

take (x+2) as common

(x+2)(3x*1 -1*1)

(x+2)(3x-1)

Find the missing side

Answers

Answer:

12.94811641 =x

Step-by-step explanation:

Since this is a right triangle, we can use trig functions

cos theta = adj/ hyp

cos 44 = x /18

18 cos 44 =x

12.94811641 =x

Answer:

12.9

Step-by-step explanation:

cos(44) = x/18

x = cos(44) * 18

x = 12.9

12
Activity 2
1. Express each as a unit rate.
a) A car travelled 150 km in 3 hour. Show your work.​

Answers

Given:

A car traveled 150 km in 3 hour.

To find:

The unit rate.

Solution:

We have,

Distance = 150 km

Time = 3 hours.

In this case speed is the unit rate.

[tex]Speed=\dfrac{Distance}{Time}[/tex]

Substituting the given values, we get

[tex]Speed=\dfrac{150\text{ km}}{3\text{ hr}}[/tex]

[tex]Speed=50\text{ km/h}[/tex]

Therefore, the unit rate is 50 km/h.

Solve for x.
2/5 ≤ x-4

Answers

2/5 ≤ x-4

0.4 ≤ x-4

let's just add 4 on both sides

0.4 +4 ≤ x-4 +4

4.4 ≤ x

simple as that. just handle inequalities like you would handle equations

PLEASE PLEASE PLEASE HELP ME!!! IM SO CLOSE TO GETTING THE SCORE I NEED

explaining your answer = brainliest

The radius of a circle is 2 kilometers. What is the area of a sector bounded by a 36° arc?

answer has to be in SIMPLEST form

Answers

Answer:

[tex]\frac{2}{5} \pi[/tex] [tex]km^{2}[/tex]

Step-by-step explanation:

In this problem, we will use the following formula:

A = [tex](\frac{C}{360}) \pi r^{2}[/tex] , where r = radius & C = central angle

Step 1: Plug in for C and r.

A =  [tex](\frac{C}{360} )\pi r^{2}[/tex]A = [tex](\frac{36}{360})\pi 2^{2}[/tex]

Step 2: Solve.

A = [tex](\frac{1}{10})4\pi[/tex]A =  [tex]\frac{2}{5} \pi[/tex]

Final answer (simplified & exact):  [tex]\frac{2}{5} \pi[/tex] [tex]km^{2}[/tex]

Sorry it took so long to complete... kinda started with the wrong answer... But I hope this helps and that you have a great rest of your day!

please help find x!! asap

Answers

Answer:

x = 60

Step-by-step explanation:

The angle 30 is on the same vertical line as the angle that is adjacent to the missing angle x. they form 90 degrees.

how do u find volume of a rectangle

Answers

Count up all the sides of the triangle and divide it by 3 if I’m
Not mistaken

Answer:

the length of the rectangle times the wedth of the rectangele by 2

Please help. Thank you. (30 points included) (Shown work)

Answers

Answer:

True

Step-by-step explanation:

Supplementary angles are angles that add up to 180 degrees (straight line). *5 degrees + x create a straight line.

Find the mode of the data.
The number of awards earned
from each school in the district.
1, 3, 1, 5, 7, 8, 10
Mode: [?]
hande in the aroon hoy

Answers

Answer:

1

Step-by-step explanation:

The mode is the number that appears most often

1 appears twice in the data list and that is the mode

Answer:

Mode: 1

Step-by-step explanation:

The mode of a data set is the number that occurs most frequently in the set. To easily find the mode, put the numbers in order from least to greatest and count how many times each number occurs. The number that occurs the most is the mode!

Simplify the expression
3(5x+8)

Answers

Answer:

Solution given:

3(5x+8)

distribute

15x+8*3

15x+24 is a required answer.

Answer:

15x + 24

Step-by-step explanation:

Apply the Distributive Property of Multiplication.  Multiply each of the two terms inside parentheses by 3:

3(5x + 8) = 15x + 24

Translate this sentence into an equation. 46 is the sum of 21 and Mabel's savings.

Answers

You have to add 21 by M to get the answer that you are looking for

Graph h(x)=-x^4 +3
Use the parabola tool then choose the vertex followed by one point on the parabola.

Answers

Hope this helps, the y intercept is 3 ans the x intercepts are +1.3 and -1.3

The y intercept is 3 and the x intercepts are +1.3 and -1.3

The graph is given below.

What is graph?

In mathematics, the graph of a function f is the set of ordered pairs, where {\displaystyle f(x)=y.} In the common case where x and f(x) are real numbers, these pairs are Cartesian coordinates of points in two-dimensional space and thus form a subset of this plane.

Here, we have,

h(x)=-x^4 +3

so, The y intercept is 3 and the x intercepts are +1.3 and -1.3

The graph is given below.

To learn more on graph click:

brainly.com/question/17267403

#SPJ2

Translate the sentence into an equation. Twice the difference of a number and 6 is 2.

Answers

Answer:

let the number be x.

2(x-6)=2

hope it helps

stay safe healthy and happy....

In the system of equations , x represents .... ( please answer 12a and 12b )

Answers

Answer:

x = cost of each t-shirt

y = cost of each sweatshirt

Step-by-step explanation:

Given equation:

2x + 2y = 26 (1)

x + 3y = 33 (2)

From (2)

x = 33 - 3y

Substitute x = 33 - 3y into (1)

2x + 2y = 26 (1)

2(33 - 3y) + 2y = 26

66 - 6y + 2y = 26

- 6y + 2y = 26 - 66

-4y = -40

y = -40/-4

y = 10

Substitute y = 10 into (2)

x + 3y = 33 (2)

x + 3(10) = 33

x + 30 = 33

x = 33 - 30

x = 3

Where,

x = cost of each t-shirt = $3

y = cost of each sweatshirt = $10

Jane and Tannic split the cost of buying a video game. The game costs$25, and the sales is 6% markup. How much do they each pay?

Answers

Answer:

$13.25

Step-by-step explanation:

Given data

Cost of game = $25

Markup= 6%

Let us find the total cost of the game

First, the markup cost

=6/100*25

=0.06*25

=$1.5

Now the total cost is

=25+1.5

=$26.5

So, each is expected to pay

=26.5/2

=$13.25

need help with thisss

Answers

9514 1404 393

Answer:

  cos(x) = 0.6

Step-by-step explanation:

First, we need to find UQ.

  UQ = UP·cos(y°) = UP·√(1 -sin²(y°)) = (26 cm)√(1 -25/169) = (26 cm)(12/13)

  UQ = 24 cm

Then TQ is ...

  TQ = 1/3UQ = 1/3(24 cm) = 8 cm

And cos(x°) is ...

  cos(x°) = √(1 -sin²(x)) = √(1 -64/100)

  cos(x°) = 6/10 = 3/5

  cos(x°) = 3/5 = 0.6

plssplsplsplspls answer fast bc its a map and I can't wait for way too long

Answers

Answer:

Part A: 1

Part B: -4

Step-by-step explanation:

In a coordinate [tex](x,y)[/tex], the x-coordinate represents the input of a function and the y-coordinate represents the output.

Part A:

We're looking for the point the line passes through with an x-coordinate (input) of -3. This point is (-3,1) and therefore the output is 1 when the input is -3.

Part B:

We're looking for the point the line passes through with a y-coordinate (output) of 2. This point is (-4,2) and therefore an input of -4 yields an output of 2.

Answer:

A) 1

B -4

well, need to write at least 20 characters... have a nice day :)

The temperature in Chicago, Illinois dropped to -9 °F during the night.
How many degrees must the temperature increase to reach 0 °F?

Answers

Answer:

+9

Step-by-step explanation:

+9-9=0

The Correct answer to your question is 0!!

4x² + 3 = -7x
solve for x
x=_,_​

Answers

[tex]\displaystyle\bf 4x^2+3=-7x\\\\4x^2+7x+3=0 \\D=49-48=1 \\\\x_{1;2}=\frac{-7\pm1}{8} => x_1=-1\quad ; \quad x_2=-\frac{3}{4} \\\\\\Answer:\boxed{x_1=-1\quad ; \quad x_2=-\frac{3}{4} }[/tex]

Other Questions
In 2011, there were 150 participants in a quiz competition. In 2012, there were 185 participants in this quiz competition.Select the true statement about participants in the quiz competition from 2011 to 2012. Percentages are rounded to the nearest tenths place.a.) The absolute change is 35 students and the relative change indicates an increase of 18.92%. b.) The absolute change is -35 students and the relative change indicates a decrease of 23.3%. c.) The absolute change is 35 students and the relative change indicates an increase of 23.3%. d.) The absolute change is -35 students and the relative change indicates a decrease of 35%. Hi, please help me it's reading in Spanish. Please I'm begging you to help me because I only have a week left for school to finish and I want to pass this class. If anyone is fluent and knows how to read in Spanish. Please help me. NO LINK!!! Also, I'll give brainiliest as well. Please help!!For the final, you will need to identify a rule that you do not like in your favorite sport and come up with a solution to improve or replace that rule. If you "do not like any sports", you can think back to a sport that we played in class (basketball, soccer, softball, and flag football) for ideas Please help me with this question How is the enthalpy of reaction shown in this potential energy diagram I am trying to graph y=2x-7 which of the following was NOT established in Article II of the Constitution?A. It vested the power to execute laws in a presidentB. It set the presidents term at four yearsC. It states the qualifications for officeD. It provided methods for selecting representatives and senators can u pls answer 22, 23, 24 The base of the flower where it is attached to the rest of the plant is the:1)pistil2)receptacle3)fruit Determine the mass in grams of hydrogen gas produced if 40.08 g of calcium react with excess aqueous hydrogen phosphate. Marking brainliest Ayuda!!! EllusFind the range of the data.The ages of people eatingchocolate ice cream at a park.12, 2, 8, 5, 3, 2,5Range: [? ]Enter the number that belongs in the green box. Which best describes the electric field created by a positive charge?A. It circles clockwise around the charge.B. It circles counterclockwise around the charge.C. Its rays point away from the charge.D. Its rays point toward the charge. Select 3 sides that can form a right triangle. Which value of n would make 3n=8 Brainliest goes to whoever answers correctly I have other questions if you want more points Consider the following argument from analogy. According our rules for appraising analogical reasoning, if a subsequent consideration strengthens the argument, answer a. Answer b if a consideration weakens the argument. Answer c if a consideration does not affect the argument.Bill has taken three history courses and found them very stimulating and valuable. So he signs up for another one, confidently expecting that it too will be worthwhile.25. Suppose that his previous history courses were in ancient history, modern European history, and American history.26. Suppose that his previous history courses had all been taught by the same professor that is scheduled to teach the present one.27. Suppose that his previous history courses all had been taught by Professor Smith, and the present one is taught by professor Jones.28. Suppose that Bill had found his three previous history courses to be the most exciting intellectual experiences of his life.29. Suppose that his previous history courses had all met at 9:00am, and the present one is scheduled to meet at 9:00am also.30. Suppose that, in addition to the three history courses previously taken, Bill also had taken and enjoyed courses in anthropology, economics, political science, and sociology. Possible answers, 35, 30, 90,64 What was one goal of the Populist Party?O A. The establishment of Grange Halls in every cityB. A lower number of immigrants allowed into the United StatesC. Getting the federal government to embrace the gold standardO D. The establishment of a national bank that would make low-interestloans to farmers. Use the following list of balances for Paul's Guitar Shop to answer the following question. Guitar Sales 24,800, Depreciation expense 2,000, Wage expense 10,950, Rent 500, Music Lesson Revenue 3,000, Interest expense 500, Supplies expense 500, Utilities expense 400. What is the TOTAL value of EXPENSES? *